Difference between revisions of "2008 AMC 12A Problems/Problem 9"

(Duplicate, AMC 10 box, style, answer choices)
m (minor edit)
 
(One intermediate revision by one other user not shown)
Line 1: Line 1:
 
{{duplicate|[[2008 AMC 12A Problems|2008 AMC 12A #9]] and [[2008 AMC 10A Problems/Problem 14|2008 AMC 10A #14]]}}
 
{{duplicate|[[2008 AMC 12A Problems|2008 AMC 12A #9]] and [[2008 AMC 10A Problems/Problem 14|2008 AMC 10A #14]]}}
==Problem==  
+
 
 +
== Problem ==  
 
Older television screens have an aspect ratio of <math>4: 3</math>. That is, the ratio of the width to the height is <math>4: 3</math>. The aspect ratio of many movies is not <math>4: 3</math>, so they are sometimes shown on a television screen by "letterboxing" - darkening strips of equal height at the top and bottom of the screen, as shown. Suppose a movie has an aspect ratio of <math>2: 1</math> and is shown on an older television screen with a <math>27</math>-inch diagonal. What is the height, in inches, of each darkened strip?
 
Older television screens have an aspect ratio of <math>4: 3</math>. That is, the ratio of the width to the height is <math>4: 3</math>. The aspect ratio of many movies is not <math>4: 3</math>, so they are sometimes shown on a television screen by "letterboxing" - darkening strips of equal height at the top and bottom of the screen, as shown. Suppose a movie has an aspect ratio of <math>2: 1</math> and is shown on an older television screen with a <math>27</math>-inch diagonal. What is the height, in inches, of each darkened strip?
<asy>unitsize(1mm);
+
 
 +
<asy>
 +
unitsize(1mm);
 
filldraw((0,0)--(21.6,0)--(21.6,2.7)--(0,2.7)--cycle,grey,black);
 
filldraw((0,0)--(21.6,0)--(21.6,2.7)--(0,2.7)--cycle,grey,black);
 
filldraw((0,13.5)--(21.6,13.5)--(21.6,16.2)--(0,16.2)--cycle,grey,black);
 
filldraw((0,13.5)--(21.6,13.5)--(21.6,16.2)--(0,16.2)--cycle,grey,black);
draw((0,0)--(21.6,0)--(21.6,16.2)--(0,16.2)--cycle);</asy>
+
draw((0,0)--(21.6,0)--(21.6,16.2)--(0,16.2)--cycle);
 +
</asy>
 +
 
 
<math>\mathrm{(A)}\ 2\qquad\mathrm{(B)}\ 2.25\qquad\mathrm{(C)}\ 2.5\qquad\mathrm{(D)}\ 2.7\qquad\mathrm{(E)}\ 3</math>
 
<math>\mathrm{(A)}\ 2\qquad\mathrm{(B)}\ 2.25\qquad\mathrm{(C)}\ 2.5\qquad\mathrm{(D)}\ 2.7\qquad\mathrm{(E)}\ 3</math>
  
==Solution==  
+
== Solution ==  
 
Let the width and height of the screen be <math>4x</math> and <math>3x</math> respectively, and let the width and height of the movie be <math>2y</math> and <math>y</math> respectively.  
 
Let the width and height of the screen be <math>4x</math> and <math>3x</math> respectively, and let the width and height of the movie be <math>2y</math> and <math>y</math> respectively.  
  
Line 17: Line 22:
 
Thus, the height of each strip is <math>\frac{3x-y}{2}=\frac{3x-2x}{2}=\frac{x}{2}=\frac{27}{10}=2.7\Longrightarrow\mathrm{(D)}</math>.  
 
Thus, the height of each strip is <math>\frac{3x-y}{2}=\frac{3x-2x}{2}=\frac{x}{2}=\frac{27}{10}=2.7\Longrightarrow\mathrm{(D)}</math>.  
  
==See Also==
+
== See Also ==
 
{{AMC12 box|year=2008|ab=A|num-b=8|num-a=10}}
 
{{AMC12 box|year=2008|ab=A|num-b=8|num-a=10}}
 
{{AMC10 box|year=2008|ab=A|num-b=13|num-a=15}}
 
{{AMC10 box|year=2008|ab=A|num-b=13|num-a=15}}
 +
{{MAA Notice}}

Latest revision as of 00:31, 19 October 2020

The following problem is from both the 2008 AMC 12A #9 and 2008 AMC 10A #14, so both problems redirect to this page.

Problem

Older television screens have an aspect ratio of $4: 3$. That is, the ratio of the width to the height is $4: 3$. The aspect ratio of many movies is not $4: 3$, so they are sometimes shown on a television screen by "letterboxing" - darkening strips of equal height at the top and bottom of the screen, as shown. Suppose a movie has an aspect ratio of $2: 1$ and is shown on an older television screen with a $27$-inch diagonal. What is the height, in inches, of each darkened strip?

[asy] unitsize(1mm); filldraw((0,0)--(21.6,0)--(21.6,2.7)--(0,2.7)--cycle,grey,black); filldraw((0,13.5)--(21.6,13.5)--(21.6,16.2)--(0,16.2)--cycle,grey,black); draw((0,0)--(21.6,0)--(21.6,16.2)--(0,16.2)--cycle); [/asy]

$\mathrm{(A)}\ 2\qquad\mathrm{(B)}\ 2.25\qquad\mathrm{(C)}\ 2.5\qquad\mathrm{(D)}\ 2.7\qquad\mathrm{(E)}\ 3$

Solution

Let the width and height of the screen be $4x$ and $3x$ respectively, and let the width and height of the movie be $2y$ and $y$ respectively.

By the Pythagorean Theorem, the diagonal is $\sqrt{(3x)^2+(4x)^2}=5x = 27$. So $x=\frac{27}{5}$.

Since the movie and the screen have the same width, $2y=4x\Rightarrow y=2x$.

Thus, the height of each strip is $\frac{3x-y}{2}=\frac{3x-2x}{2}=\frac{x}{2}=\frac{27}{10}=2.7\Longrightarrow\mathrm{(D)}$.

See Also

2008 AMC 12A (ProblemsAnswer KeyResources)
Preceded by
Problem 8
Followed by
Problem 10
1 2 3 4 5 6 7 8 9 10 11 12 13 14 15 16 17 18 19 20 21 22 23 24 25
All AMC 12 Problems and Solutions
2008 AMC 10A (ProblemsAnswer KeyResources)
Preceded by
Problem 13
Followed by
Problem 15
1 2 3 4 5 6 7 8 9 10 11 12 13 14 15 16 17 18 19 20 21 22 23 24 25
All AMC 10 Problems and Solutions

The problems on this page are copyrighted by the Mathematical Association of America's American Mathematics Competitions. AMC logo.png